Which one of the following, if substituted for the constraint that Flynn must be on desk duty earlier in the week tha...

FS101 on June 24, 2020

June 2015 Practice Test - Question 18 (logic games)

For answer choice C why can't you just have it: H L K M F G+Z? That still abides by the rules (excluding rule 3) so why is answer C correct if this is a possibility?

Replies
Create a free account to read and take part in forum discussions.

Already have an account? log in

shunhe on June 25, 2020

Hi @Sheikhd,

Thanks for the question! So a couple of things here: answer (C) is correct because it abides by the rules. What we’re trying to do with the rule substitution questions is put in a new rule that will result in the exact same possible distributions. So if (C) abides by the rules in the same way as the original rules, then it would be the correct answer.

That being said, if the example you came up with were true, then answer (C) would not be correct. Because the original rules say that F has to be before K and M. So if your proposed distribution of H L K M F G+Z was possible under the new rule, then (C) would be incorrect.

However, the distribution you proposed violates what answer choice (C) says. Remember, (C) says that ONLY H and L can be on desk duty earlier than F. That means H and L, and no others, can go before F. But in your proposed example, we also have K and M going before F, which violates what answer choice (C) is saying; that’s not a possible distribution under answer choice (C)’s rules. And so (C) is the correct answer.

Hope this helps! Feel free to ask any other questions that you might have.

boogity on November 7, 2020

Is there a way to answer these types of questions faster? Also, is it possibly to answer these types of questions without using the set-up methods and specific strategy of mapping out different scenarios that this course teaches?

Avery on May 24, 2022

^How do we approach these questions?

Emil-Kunkin on May 25, 2022

Hi,

These rule replacement questions do tend to be quite time consuming- but I like to use two tests as I go through the answer choices. First, if an answer does too much we can eliminate it. That is, if the new rule eliminates a scenario that would have been possible before, then we can eliminate that answer choice.

We can also eliminate answer choices that do too little. That is, they allow a violation of the initial rule. These are usually quicker ones to spot- you can see if the new rule would allow something to happen that would have been banned in the initial rule.

Finally, when the rule being replaced is conditional, you should look for the contrapositive. They will occasionally make the correct answer the contrapositive of the initial rule.

I would also recommend office hours on rule substitution questions. You can use the filters to find them.